Board logo

标题: [逻辑资料] XDF逻辑GZW老师的课堂录音以及讲义 [打印本页]

作者: wangyilin888    时间: 2011-4-27 10:49     标题: XDF逻辑GZW老师的课堂录音以及讲义

GMAT逻辑 讲义

高智威


归纳题………………………………………………… 1

削弱题………………………………………………… 12

加强题………………………………………………… 37

假设题………………………………………………… 54

解释题………………………………………………… 73

填空题………………………………………………… 83

句子作用题…………………………………………… 94

评价题………………………………………………… 106


作者: wangyilin888    时间: 2011-4-27 10:50

例题

1. Meteorite explosions in the Earth’s atmosphere as large as the one that destroyed forests in Siberia, with approximately the force of a twelve-megaton nuclear blast, occur about once a century. The response of highly automated systems controlled by complex computer programs to unexpected circumstances is unpredictable.


Which of the following conclusions can most properly be drawn, if the statements above are true, about a highly automated nuclear-missile defense system controlled by a complex computer program?


A. Within a century after its construction, the system would react inappropriately and might accidentally start a nuclear war.

B. The system would be destroyed if an explosion of a large meteorite occurred in the Earth’s atmosphere.

C. It would be impossible for the system to distinguish the explosion of a large meteorite from the explosion of a nuclear weapon.

D. Whether the system would respond inappropriately to the explosion of a large meteorite would depend on the location of the blast.

E. It is not certain what the system’s response to the explosion of a large meteorite would be, if its designers did not plan for such a contingency.


作者: wangyilin888    时间: 2011-4-27 10:50

2.
The fewer restrictions there are on the advertising of legal services, the more lawyers there are who advertise their services, and the lawyers who advertise a specific service usually charge less for that service than lawyers who do not advertise. Therefore, if the state removes any of its current restrictions, such as the one against advertisements that do not specify fee arrangements, overall consumer legal costs will be lower than if the state retains its current restrictions.


If the statements in the passage are true, which of the following must be true?


A. Some lawyers who now advertise will charge more for specific services if they do not have to specify fee arrangements in the advertisements.

B. More consumers will use legal services if there are fewer restrictions on the advertising of legal service.

C. If the restriction against advertisements that do not specify fee arrangements is removed, more lawyers will advertise their services.

D. If more lawyers advertise lower prices for specific services, some lawyers who do not advertise will also charge less than they currently charge for those services.

E. If the only restrictions on the advertising of legal services were those that apply to every type of advertising, most lawyers would advertise their services.


作者: wangyilin888    时间: 2011-4-27 10:51

3. Rural households have more purchasing power than do urban or suburban households at the same income level, since some of the income urban and suburban households use for food and shelter can be used by rural households for other needs.

Which of the following inferences is best supported by the statement made above?

A. The average rural household includes more people than does the average urban or suburban household.
B. Rural households have lower food and housing costs than do either urban or suburban households.
C. Suburban households generally have more purchasing power than do either rural or urban households.
D. The median income of urban and suburban households is generally higher than that of rural households.
E. All three types of households spend more of their income on housing than on all other purchases combined.
作者: wangyilin888    时间: 2011-4-27 10:51

4. When a polygraph test is judged inconclusive, this is no reflection on the examinee. Rather, such a judgment means that the test has failed to show whether the examinee was truthful or untruthful. Nevertheless, employers will sometimes refuse to hire a job applicant because of an inconclusive polygraph test result.
Which of the following conclusions can most properly be drawn from the information above?

A. Most examinees with inconclusive polygraph test results are in fact untruthful.
B. Polygraph tests should not be used by employers in the consideration of job applicants.
C. An inconclusive polygraph test result is sometimes unfairly held against the examinee.
D. A polygraph test indicating that an examinee is untruthful can sometimes be mistaken.
E. Some employers have refused to consider the results of polygraph tests when evaluating job applicants.
作者: wangyilin888    时间: 2011-4-27 10:52

5. (Z1) That the application of new technology can increase the productivity of existing coal mines is demonstrated by the case of Tribnia’s coal industry.  Coal output per miner in Tribnia is double what it was five years ago, even though no new mines have opened.

Which of the following can be properly concluded from the statement about coal output per miner in the passage?

A.        If the number of miners working in Tribnian coal mines has remained constant in the past five years, Tribnia’s total coal production has doubled in that period of time.
B.        Any individual Tribnian coal mine that achieved an increase in overall output in the past five years has also experienced an increase in output per miner.
C.        If any new coal mines had opened in Tribnia in the past five years, then the increase in output per miner would have been even greater than it actually was.
D.        If any individual Tribnian coal mine has not increased its output per miner in the past five years, then that mine’s overall output has declined or remained constant.
E.        In Tribnia the cost of producing a given quantity of coal has declined over the past five years.
作者: wangyilin888    时间: 2011-4-27 10:53

6. (Z2-20) Shipping Clerk:  The five specially ordered shipments sent out last week were sent out on Thursday.  Last week, all of the shipments that were sent out on Friday consisted entirely of building supplies, and the shipping department then closed for the weekend.  Four shipments were sent to Truax Construction last week, only three of which consisted of building supplies.

If the shipping clerk’s statements are true, which of the following must also be true?

A.        At least one of the shipments sent to Truax Construction last week was specially ordered.
B.        At least one of last week’s specially ordered shipments did not consist of building supplies.
C.        At least one of the shipments sent to Truax Construction was not sent out on Thursday of last week.
D.        At least one of the shipments that were sent out on Friday of last week was sent to Truax Construction.
E.        At least one of the shipments sent to Truax Construction last week was sent out before Friday.
作者: wangyilin888    时间: 2011-4-27 10:53

7. (Z2-30) Although most smoking-related illnesses are caused by inhaling the tar in tobacco smoke, it is addiction to nicotine that prevents most smokers from quitting.  In an effort to decrease the incidence of smoking-related illnesses, lawmakers in Sandonia plan to reduce the average quantity of nicotine per cigarette by half over the next five years.  Unfortunately, smokers who are already addicted to nicotine tend to react to such reductions by smoking correspondingly more cigarettes.

The information above most strongly supports which of the following predictions about the effects of implementing the Sandonian government’s plan?

A.        The average quantity of tar inhaled by Sandonian smokers who are currently addicted to nicotine will probably not decrease during the next five years.
B.        Sandonian smokers who are not already addicted to nicotine will probably also begin to smoke more cigarettes during the next five years than they had previously.
C.        The annual number of Sandonian smokers developing smoking-related illnesses will probably decrease during the next five years.
D.        The proportion of Sandonians attempting to quit smoking who succeed in that attempt will probably decrease during the next five years.
E.        The number of Sandonians who quit smoking during the next five years will probably exceed the number who quit during the last five years.
作者: wangyilin888    时间: 2011-4-27 10:53

8. (Z3-32) From 1980 to 1989, total consumption of fish in the country of Jurania increased by 4.5 percent, and total consumption of poultry products there increased by 9.0 percent.  During the same period, the population of Jurania increased by 6 percent, in part due to immigration to Jurania from other countries in the region.

If the statements above are true, which of the following must also be true on the basis of them?

A. During the 1980’s in Jurania, profits of wholesale distributors of poultry products increased at a greater rate than did profits of wholesale distributors of fish.
B. For people who immigrated to Jurania during the 1980’s, fish was less likely to be a major part of their diet than was poultry.
C. In 1989 Juranians consumed twice as much poultry as fish.
D. For a significant proportion of Jurania’s population, both fish and poultry products were a regular part of their diet during the 1980’s.
E. Per capita consumption of fish in Jurania was lower in 1989 than in 1980.
作者: wangyilin888    时间: 2011-4-27 10:54

9. A company's two divisions performed with remarkable consistency over the past three years: in each of those years, the pharmaceuticals division has accounted for roughly 20 percent of dollar sales and 40 percent of profits, and the chemicals division for the balance.

Regarding the past three years, which of the following can properly be inferred from the statement above?

(A)        Total dollar sales for each of the company's divisions have remained roughly constant.
(B)        The pharmaceuticals division has faced stiffer competition in its markets than has the chemicals division.
(C)        The chemicals division has realized lower profits per dollar of sales than has the pharmaceuticals division.
(D)        The product mix offered by each of the company's divisions has remained unchanged.
(E)        Highly profitable products accounted for a higher percentage of the chemicals division's sales than of the pharmaceuticals division's.
作者: wangyilin888    时间: 2011-4-27 10:55

10.(Z6-40) From 1973 to 1976, total United States consumption of cigarettes increased 3.4 percent, and total sales of chewing tobacco rose 18.0 percent. During the same period, total United States population increased 5.0 percent.

If the statements above are true, which of the following conclusions can be properly drawn?

A.        United States manufacturers of tobacco products had higher profits in 1976 than in 1973.
B.        Per capita consumption of cigarettes in the United States was lower in 1976 than in 1973.
C.        The proportion of nonsmokers in the United States population dropped slightly between 1973 and 1976.
D.        United States manufacturers of tobacco products realize a lower profit on cigarettes than on chewing tobacco.
E.        A large percentage of United States smokers switched from cigarettes to chewing tobacco between 1973 and 1976.

例题答案:ECBCA EAECB
作者: wangyilin888    时间: 2011-4-27 10:55

练习题:
1. Advertisement: Today's customers expect high quality. Every advance in the quality of manufactured products raises customer expectations. The company that is satisfied with the current quality of its products will soon find that its customers are not. At MegaCorp, meeting or exceeding customer expectations is our goal.

Which of the following must be true on the basis of the statements in the advertisement above?

(A)        MegaCorp's competitors will succeed in attracting customers only if those competitors adopt MegaCorp's goal as their own.
(B)        A company that does not correctly anticipate the expectations of its customers is certain to fail in advancing the quality of its products.
(C)        MegaCorp's goal is possible to meet only if continuing advances in product quality are possible.
(D)        If a company becomes satisfied with the quality of its products, then the quality of its products is sure to decline.
(E)        MegaCorp's customers are currently satisfied with the quality of its products.
作者: wangyilin888    时间: 2011-4-27 10:56

2. Social scientists are underrepresented on the advisory councils of the National Institutes of Health (NIH). Since these councils advise NIH directors and recommend policy, the underrepresentation of social scientists results in a relative lack of NIH financial support for research in the social sciences.
If the statements above are correct, they most strongly support which of the following?
(A)        A significant increase in the size of NIH advisory councils would be required in order to increase the representation of social scientists on these councils.
(B)        A significant increase in the representation of social scientists on NIH advisory councils would result in an increase in NIH funding for social science research.
(C)        A significant increase in funding for social science research would result in improved policy' recommendations to NIH directors.
(D)        A significant increase in funding for the training of social scientists would result in an increase in the number of social scientists on NIH advisory councils.
(E)        A significant increase in the representation of social scientists on NIH advisory councils would have to precede any increase in the number of NIH directors who are social scientists.
作者: wangyilin888    时间: 2011-4-27 10:57

3.(Z7-13)Fact: Asthma, a bronchial condition, is much less common ailment than hay fever, an allergic inflammation of the nasal passages.

Fact: over 95 percent of people who have asthma also suffer from hay fever.

If the information given as facts above is true, which of the following must also be true?

A.        Hay fever is a prerequisite for the development of asthma
B.        Asthma is a prerequisite for the development of hay fever
C.        Those who have neither hay fever nor asthma comprise less than 5 percent of the total population
D.        The number of people who have both of these ailments is greater than the number of people who have only one of them  
E.        The percentage of people suffering from hay fever who also have asthma is lower than 95 percent.
作者: wangyilin888    时间: 2011-4-27 10:58

4. (Z8-30)The quality of unrefined olive oil is not actually defined in terms of acidity, yet extensive tests have shown that the less free oleic acid an unrefined olive oil contains per liter, the higher its quality.  The proportion of free oleic acid that an olive oil contains is an accurate measure of the oil’s acidity.

If the statements above are all true, which of the following conclusions is best supported by them?

A.        When an olive oil is refined, the concentration of oleic acid in the oil is reduced.
B.        The quality of an unrefined olive oil can be determined only by accurately measuring its acidity.
C.        If an unrefined olive oil is intermediate in acidity between two other unrefined olive oils, it will also be intermediate between them in quality.
D.        Free oleic acid is the only acid that unrefined olive oil contains.
E.        People who judge the quality of unrefined olive oils actually judge those oils by their acidity, which the judges can taste.
作者: wangyilin888    时间: 2011-4-27 10:59

5.(Z11-23)Macrophages are cells that play a role in the response of the immune system of mice and other mammals to invasive organisms such as bacteria.  Unlike other mice, mice that are genetically incapable of making these particular cells do not show elevated levels of nitrates when infected with bacteria.

The statements above, if true, provide the most support for which of the following conclusions?

A.        Mice that are unable either to make macrophages or to make them in sufficient numbers will protect themselves from bacterial infections in some other way.
B.        Mice that show elevated levels of nitrates can easily fight off most types of bacterial infections.
C.        In mice, macrophages play a role in the production of nitrates or inhibit a process by which nitrates are broken down or otherwise eliminated.
D.        When a healthy mouse becomes infected with an invasive organism, the number of macrophages in the mouse’s body decreases.
E.        Injections of nitrates into mice that lack macrophages will not enhance the ability of these animals’ immune systems to fight off infection.
作者: wangyilin888    时间: 2011-4-27 10:59

6.(Z13-30) The percentage of households with an annual income of more than $40,000 is higher in Merton county than in any other county. However, the percentage of households with an annual income of $60,000 or more is highest in Sommer county.

If the statements above are true, which of the following can properly be concluded on the basis of them?

A.        No households in Merton county has an annul income of $60,000 or more
B.        Some households in Merton county have an annual income between $40,000 and $60,000
C.        The number of households with an annual income of more than $40,000 is greater in Merton than in Sommer county.
D.        Average annual household income is higher in Sommer than in Merton county.
E.        The percentage of households with an annual income of $80,000 is higher in Sommer than in Merton county.
作者: wangyilin888    时间: 2011-4-27 11:00

7. Among the more effective kinds of publicity that publishers can get for a new book is to have excerpts of it published in a high-circulation magazine soon before the book is published. The benefits of such excerption include not only a sure increase in sales but also a fee paid by the magazine to the book's publisher.

Which of the following conclusions is best supported by the information above?

(A)        The number of people for whom seeing an excerpt of a book in a magazine provides an adequate substitute for reading the whole book is smaller than the number for whom the excerpt stimulates a desire to read the book.
(B)        Because the financial advantage of excerpting a new book in a magazine usually accrues to the book's publisher, magazine editors are unwilling to publish excerpts from new books.
(C)        In calculating the total number of copies that a book has sold, publishers include sales of copies of magazines that featured an excerpt of the book.
(D)        The effectiveness of having excerpts of a book published in a magazine, measured in terms of increased sales of a book, is proportional to the circulation of the magazine in which the excerpts are published.
(E)        Books that are suitable for excerpting in high-circulation magazines sell more copies than books that are not suitable for excerpting.
作者: wangyilin888    时间: 2011-4-27 11:03

8. The fewer restrictions there are on the advertising of legal services, the more lawyers there are who advertise their services, and the lawyers who advertise a specific service usually charge less for that service than the lawyers who do not advertise. Therefore, if the state removes any of its current restrictions, such as the one against advertisements that do not specify fee arrangements, overall consumer legal costs will be lower than if the state retains its current restrictions.

If the statements above are true, which of the following must be true?

(A)        Some lawyers who now advertise will charge more for specific services if they do not have to specify fee arrangements in the advertisements.
(B)        More consumers will use legal services if there are fewer restrictions on the advertising of legal services.
(C)        If the restriction against advertisements that do not specify fee arrangements is removed, more lawyers will advertise their services.
(D)        If more lawyers advertise lower prices for specific services, some lawyers who do not advertise will also charge less than they currently charge for those services.
(E)        If the only restrictions on the advertising of legal services were those that apply to every type of advertising, most lawyers would advertise their services.
作者: wangyilin888    时间: 2011-4-27 11:03

9.To protect certain fledgling industries, the government of Country Z banned imports of the types of products those industries were starting to make. As a direct result, the cost of those products to the buyers, several export-dependent industries in Z, went up, sharply limiting the ability of those industries to compete effectively in their export markets.

Which of the following can be most properly inferred from the passage about the products whose importation was banned?

(A)        Those products had been cheaper to import than they were to make within Country Z's fledgling industries.
(B)        Those products were ones that Country Z was hoping to export in its turn, once the fledgling industries matured.
(C)        Those products used to be imported from just those countries to which Country Z's exports went.
(D)        Those products had become more and more expensive to import, which resulted in a foreign trade deficit just before the ban.
(E)        Those products used to be imported in very small quantities, but they were essential to Country Z's.
作者: wangyilin888    时间: 2011-4-27 11:03

10. Although aspirin has been proven to eliminate moderate fever associated with some illnesses, many doctors no longer routinely recommend its use for this purpose. A moderate fever stimulates the activity of the body's disease-fighting white blood cells and also inhibits the growth of many strains of disease-causing bacteria.

If the statements above are true, which of the following conclusions is most strongly supported by them?

A. Aspirin, an effective painkiller, alleviates the pain and discomfort of many illnesses.
B. Aspirin can prolong a patient's illness by eliminating moderate fever helpful in fighting some diseases.
C. Aspirin inhibits the growth of white blood cells, which are necessary for fighting some illnesses.
D. The more white blood cells a patient's body produces, the less severe the patient's illness will be.
E. The focus of modern medicine is on inhibiting the growth of disease-causing bacteria within the body.
作者: wangyilin888    时间: 2011-4-27 11:04

11. Roland: The alarming fact is that 90 percent of the people in this country now report that they know someone who is unemployed.
Sharon: But a normal, moderate level of unemployment is 5 percent, with one out of 20 workers unemployed. So at any given time if a person knows approximately 50 workers, one or more will very likely be unemployed.

Sharon's argument is structured to lead to which of the following as a conclusion?

(A)        The fact that 90 percent of the people know someone who is unemployed is not an indication that unemployment is abnormally high.
(B)        The current level of unemployment is not moderate.
(C)        If at least 5 percent of workers are unemployed, the result of questioning a representative group of people cannot be the percentage Roland cites.
(D)        It is unlikely that the people whose statements Roland cites are giving accurate reports.
(E)        If an unemployment figure is given as a certain percentage, the actual percentage of those without jobs is even higher.
作者: wangyilin888    时间: 2011-4-27 11:04

12. Environmentalist: The commissioner of the Fish and Game Authority would have the public believe that increases in the number of marine fish caught demonstrate that this resource is no longer endangered. This
is a specious argument, as unsound as it would be to assert that the ever-increasing rate at which rain forests are being cut down demonstrates a lack of danger to that resource. The real cause of the increased fish-catch is a greater efficiency in using technologies that deplete resources.

The environmentalist's statements, if true, best support which of the following as a conclusion?

(A)        The use of technology is the reason for the increasing encroachment of people on nature.
(B)        It is possible to determine how many fish are in the sea by some.way other than by catching fish.
(C)        The proportion of marine fish that are caught is as high as the proportion of rain forest trees that are cut down each year.
(D)        Modern technologies waste resources by catching inedible fish.
(E)        Marine fish continue to be an endangered resource.
作者: wangyilin888    时间: 2011-4-27 11:04

2. 削弱题
演绎题包括:

演绎题“题目”类型:





演绎题“题目”阅读练习:
        In recent years, many cabinetmakers have been winning acclaim as artists. But since furniture must be useful, cabinetmakers must exercise their craft with an eye to the practical utility of their product. For this reason, cabinetmaking is not art.

总结:________________________________________________________

        Within 20 years it will probably be possible to identify the genetic susceptibility an individual may have toward any particular disease. Eventually, effective strategies will be discovered to counteract each such susceptibility. Once these effective strategies are found, therefore, the people who follow them will never get sick.

总结:________________________________________________________
作者: wangyilin888    时间: 2011-4-27 11:05

        The technological conservatism of bicycle manufacturers is a reflection of the kinds of demand they are trying to meet. The only cyclists seriously interested in innovation and willing to pay for it are bicycle racers. Therefore, innovation in bicycle technology is limited by what authorities will accept as standard for purposes of competition in bicycle races.   

总结:________________________________________________________
         

        Red blood cells in which the malarial-fever parasite resides are eliminated from a person's body after 120 days. Because the parasite cannot travel to a new generation of red blood cells, any fever that develops in a person more than 120 days after that person has moved to a malaria-free region is not due to the malarial parasite.

总结:________________________________________________________

        In the aftermath of a worldwide stock-market crash, Country T claimed that the severity of the stock-market crash it experienced resulted from the accelerated process of denationalization many of its industries underwent shortly before the crash.

总结:________________________________________________________
        Mice that have been given morphine are very likely to develop blood poisoning because bacteria that normally reside in the intestine typically respond to morphine by migrating into the bloodstream.  However, when mice are given both morphine and the new drug naltrexone, blood poisoning is much less frequent, although it does still occur.  These results provide support for researchers’ prediction that naltrexone will turn out to be toxic to certain types of bacteria.

总结:________________________________________________________
作者: wangyilin888    时间: 2011-4-27 11:05

削弱题标志词:


削弱题类型:







削弱题注意事项:





例题
1. Homeowners aged 40 to 50 are more likely to purchase ice cream and are more likely to purchase it in larger amounts than are members of any other demographic group. The popular belief that teenagers eat more ice cream than adults must, therefore, be false.

The argument is flawed primarily because the author

A. Fails to distinguish between purchasing and consuming
B. Does not supply information about homeowners in age groups other than 40 to 50
C. Depends on popular belief rather than on documented research findings
D. Does not specify the precise amount of ice cream purchased by any demographic group
E. Discusses ice cream rather than more nutritious and healthful foods
作者: wangyilin888    时间: 2011-4-27 11:05

2. Opponents of laws that require automobile drivers and passengers to wear seat belts argue that in a free society people have the right to take risks as long as the people do not harm other as a result of taking the risks. As a result, they conclude that it should be each person’s decision whether or not to wear a seat belt.

Which of the following, if true, most seriously weakens the conclusion drawn above?

A. Many new cars are built with seat belts that automatically fasten when someone sits in the front seat.
B. Automobile insurance rates for all automobile owners are higher because of the need to pay for the increased injuries or deaths of people not wearing seat belts.
C. Passengers in airplanes are required to wear seat belts during takeoffs and landings.
D. The rate of automobile fatalities in states that do not have mandatory seat belt laws is greater than the rate of fatalities in states that do have such laws.
E. In automobile accidents, a greater number of passengers who do not wear seat belts are injured than are passengers who do wear seat belts.
作者: wangyilin888    时间: 2011-4-27 11:06

3. Offshore oil-drilling operations entail an unavoidable risk of an oil spill, but importing oil on tankers presently entails an even greater such risk per barrel of oil. Therefore, if we are to reduce the risk of an oil spill without curtailing our use of oil, we must invest more in offshore operations and import less oil on tankers.

Which of the following, if true, most seriously weakens the argument above?

A. Tankers can easily be redesigned so that their use entails less risk of an oil spill.
B. Oil spills caused by tankers have generally been more serious than those caused by offshore operations.
C. The impact of offshore operations on the environment can be controlled by careful management.
D. Offshore operations usually damage the ocean floor, but tankers rarely cause such damage.
E. Importing oil on tankers is currently less expensive than drilling for it offshore.
作者: wangyilin888    时间: 2011-4-27 11:06

4. The average normal infant born in the United States weighs between 12 and 14 pounds at the age of three months. Therefore, if a three-month-old child weighs only 10 pounds, its weight gain has been below the United States average.

Which of the following indicates a flaw in the reasoning above?
       
A.        Weight is only one measure of normal infant development.
B.        Some three-month-old children weigh as much as 17 pounds.
C.        It is possible for a normal child to weigh 10 pounds at birth.
D.        The phrase "below average" does not necessarily mean insufficient.
E.        Average weight gain is not the same as average weight.
作者: wangyilin888    时间: 2011-4-27 11:06

5. (Z4-20)Five years ago, as part of a plan to encourage citizens of Levaska to increase the amount of money they put into savings, Levaska’s government introduced special savings accounts in which up to $3,000 a year can be saved with no tax due on the interest unless money is withdrawn before the account holder reaches the age of sixty-five.  Millions of dollars have accumulated in the special accounts, so the government’s plan is obviously working.

Which of the following, if true, most seriously weakens the argument?

A.        A substantial number of Levaskans have withdrawn at least some of the money they had invested in the special accounts.
B.        Workers in Levaska who already save money in long-term tax-free accounts that are offered through their workplace cannot take advantage of the special savings accounts introduced by the government.
C.        The rate at which interest earned on money deposited in regular savings accounts is taxed depends on the income bracket of the account holder.
D.        Many Levaskans who already had long-term savings have steadily been transferring those savings into the special accounts.
E.        Many of the economists who now claim that the government’s plan has been successful criticized it when it was introduced.
作者: wangyilin888    时间: 2011-4-27 11:06

6. (Z4-21)An overwhelming proportion of the most productive employees at SaleCo’s regional offices work not eight hours a day, five days a week, as do other SaleCo employees, but rather ten hours a day, four days a week, with Friday off.  Noting this phenomenon, SaleCo’s president plans to increase overall productivity by keeping the offices closed on Fridays and having all employees work the same schedule—ten hours a day, four days a week.

Which of the following, if true, provides the most reason to doubt that the president’s plan, if implemented, will achieve its stated purpose?

A.        Typically, a SaleCo employee’s least productive hours in the workplace are the early afternoon hours.
B.        None of the employees who work four days a week had volunteered to work that schedule, but all were assigned to it by their supervisors.
C.        Working ten hours a day has allowed the most productive employees to work two hours alone each day in their respective offices relatively undisturbed by fellow employees.
D.        Employees at SaleCo are compensated not on the basis of how many hours a week they work but on the basis of how productive they are during the hours they are at work.
E.        Those SaleCo employees who have a four-day workweek do not take any of their office work to do at home on Fridays.
作者: wangyilin888    时间: 2011-4-27 11:07

7.(Z3-15) Lightbox, Inc., owns almost all of the movie theaters in Washington County and has announced plans to double the number of movie screens it has in the county within five years.  Yet attendance at Lightbox’s theaters is only just large enough for profitability now and the county’s population is not expected to increase over the next ten years.  Clearly, therefore, if there is indeed no increase in population, Lightbox’s new screens are unlikely to prove profitable.

Which of the following, if true about Washington County, most seriously weakens the argument?

A.        Though little change in the size of the population is expected, a pronounced shift toward a younger, more affluent, and more entertainment-oriented population is expected to occur.
B.        The sales of snacks and drinks in its movie theaters account for more of Lightbox’s profits than ticket sales do.
C.        In selecting the mix of movies shown at its theaters, Lightbox’s policy is to avoid those that appeal to only a small segment of the moviegoing population.
D.        Spending on video purchases, as well as spending on video rentals, is currently no longer increasing.
E.        There are no population centers in the county that are not already served by at least one of the movie theaters that Lightbox owns and operates.
作者: wangyilin888    时间: 2011-4-27 11:07

8.(Z3-28)Wind farms, which generate electricity using arrays of thousands of wind-powered turbines, require vast expanses of open land.  County X and County Y have similar terrain, but the population density of County X is significantly higher than that of County Y.  Therefore, a wind farm proposed for one of the two counties should be built in County Y rather than in County X.

Which of the following, if true, most seriously weakens the planner’s argument?

A.        County X and County Y are adjacent to each other, and both are located in the windiest area of the state.
B.        The total population of County Y is substantially greater than that of County X.
C.        Some of the electricity generated by wind farms in County Y would be purchased by users outside the county.
D.        Wind farms require more land per unit of electricity generated than does any other type of electrical-generation facility.
E.        Nearly all of County X’s population is concentrated in a small part of the county, while County Y’s population is spread evenly throughout the country.
作者: wangyilin888    时间: 2011-4-27 11:07

9. (Z5)Of patients over 65 years old who survived coronary bypass surgery—a procedure widely prescribed for people with heart disease—only 75 percent benefited from the surgery.  Thus it appears that for one in four such patients, the doctors who advised them to undergo this surgery, with its attendant risks and expense, were more interested in an opportunity to practice their skills and in their fee than in helping the patient.

Which of the following, if true, most seriously undermines the argument?

A.        Many of the patients who receive coronary bypass surgery are less than 55 years old.
B.        Possible benefits of coronary bypass surgery include both relief from troubling symptoms and prolongation of life.
C.        Most of the patients in the survey decided to undergo coronary bypass surgery because they were advised that the surgery would reduce their risk of future heart attacks.
D.        The patients over 65 years old who did not benefit from the coronary bypass surgery were as fully informed as those who did benefit from the surgery as to the risks of the surgery prior to undergoing it.
E.        The patients who underwent coronary bypass surgery but who did not benefit from it were medically indistinguishable, prior to their surgery, from the patients who did benefit.
作者: wangyilin888    时间: 2011-4-27 11:08

10.(Z5-30)Printwell’s Ink Jet Division manufactures ink-jet printers and the ink cartridges they use.  Sales of its ink-jet printers have increased.  Monthly revenues from those sales, however, have not increased, because competition has forced Printwell to cut the prices of its printers.  Unfortunately, Printwell has been unable to bring down the cost of manufacturing a printer.  Thus, despite the increase in printer sales, the Ink Jet Division must be providing the company with much smaller than it used to.

Which of the following, if true, most seriously weakens the argument?

A. Ink-jet printers in regular use frequently need new ink cartridges, and Printwell’s printers only accept Printwell’s ink cartridges.
B. Unlike some competing companies, Printwell sells all of its printers through retailers, and these retailers’ costs account for a sizable proportion of the printers’ ultimate retail price.
C. Some printer manufacturers have been forced to reduce the sale price of their ink-jet printers even more than Printwell has.
D. In the past year, no competing manufacturer of ink-jet printers has had as great an increase in unit sales of printers as Printwell has.
E. In the past year, sales of Printwell’s ink-jet printers have increased more than sales of any other type of printer made by Printwell.
作者: wangyilin888    时间: 2011-4-27 11:08

11.(Z7-8)Many large department stores in Montalia now provide shopping carts for their customers. Since customers using shopping carts tend to buy more than those without shopping carts, most of these stores are experiencing strong sales growth, while most other department stores are not. Therefore, in order to boost sales, managers of Jerrod’s, Montalia’s premier department store, are planning to purchase shopping carts and make them available to the store’s customers next month.

Which of the following, if true, casts most doubt whether the managers’ plan, if implemented, will achieve its goal?

A.        Since most customers associate shopping carts with low-quality discount stores, Jerrod’s high-quality image would likely suffer if shopping carts were introduced.
B.        Because the unemployment rate has declined to very low levels, Jerrod’s now has to pay significantly higher wages in order to retain its staff.
C.        A number of department stores that did not make shopping carts available to their customers have had to close recently due to falling profits.
D.        Shopping carts are not very expensive, but they generally need to be replaced every few years.
E.        Stores that make shopping carts available to their customers usually have to hire people to retrieve the carts from parking areas.
作者: wangyilin888    时间: 2011-4-27 11:08

12. (Z7-28)Editorial in Krenlandian Newspaper:
Krenland’s steelmakers are losing domestic sales because of lower-priced imports, in many cases because foreign governments subsidize their steel industries in ways that are banned by international treaties. But whatever the cause, the cost is ultimately going to be jobs in Krenland’s steel industry. Therefore, it would protect not only steel companies but also industrial employment in Krenland if our government took measures to reduce cheap steel imports.

Which of the following, if true, most seriously weakens the editorial’s argument?

A.        Because steel from Krenland is rarely competitive in international markets, only a very small portion of Krenlandian steelmakers’ revenue comes from exports.
B.        The international treaties that some governments are violating by giving subsidies to steelmakers do not specify any penalties for such violations.
C.        For many Krenlandian manufacturers who face severe international competition in both domestic and export markets, steel constitutes a significant part of their raw material costs.
D.        Because of advances in order-taking, shipping, and inventory systems, the cost of shipping steel from foreign producers to Krenland has fallen considerably in recent years.
E.        Wages paid to workers in the steel industry in Krenland differ significantly from wages paid to workers in many of the countries that export steel to Krenland.
作者: wangyilin888    时间: 2011-4-27 11:09

13.(T3-21)Driving the steep road to the mountaintop Inca ruins of Machu Picchu is potentially dangerous and hiking there is difficult. Now the Peruvian government is installing a cable car that will make access much easier, and hence results in a large increase in tourisms. However, since the presence of large numbers of tourists tends to accelerate the deterioration of a site, installation of a cable car is certain to result in harm to the ruins.

Which of the following, if true, most seriously calls into question the argument?

A.        The daily number of tourists that are expected to take the cable car to Machu Picchu is smaller than the original resident
B.        The construction of the cable car terminal at Machu Picchu will require the use of potentially damaging heavy machinery at the site.
C.        Machu Picchu is already one of the most popular tourist sites in Peru
D.        Natural weathering will continue to be a more significant cause of the deterioration of Machu Picchu than tourist traffic
E.        The cable car will replace the tour buses whose large wheels and corrosive exhaust at present do significant damage to the site.
作者: wangyilin888    时间: 2011-4-27 11:09

14.(T4-21)The OLEX Petroleum Company has recently determined that it could cut its refining costs by closing its Grenville refinery and consolidating all refining at its Tasberg refinery. Closing the Grenville refinery, however, would mean the immediate loss of about 1,2000 jobs in the Grenville area. Eventually the lives of more than 10,000 people would be seriously disrupted. Therefore, OLEX’s decision, announced yesterday, to keep Grenville open shows that at OLEX social concerns sometimes outweigh the desire for higher profits.

Which of the following, if true, most seriously undermines the argument given?

A.        The Grenville refinery, although it operates at a higher cost than the Tasberg refinery, has nevertheless been moderately profitable for many years.
B.        Even though OLEX could consolidate all its refining at the Tasberg plant, doing so at the Grenville plant would not be feasible.
C.        The Tasberg refinery is more favorably situated than the Grenville refinery with respect to the major supply routes for raw petroleum.
D.        If the Grenville refinery were ever closed and operations at the Tasberg refinery expanded, job openings at Tasberg would to the extent possible be filled with people formerly employed at Grenville.
E.        Closure of the Grenville refinery would mean compliance, at enormous cost, with demanding local codes regulating the cleanup of abandoned industrial sites.
作者: wangyilin888    时间: 2011-4-27 11:09

15. Dental researchers recently discovered that tooth-brushes can become contaminated with bacterial that cause pneumonia and strep throat. They found that contamination usually occurs after toothbrushes have been used for four weeks. For that reason, people should replace their toothbrushes at least once a month.

Which of the following, if true, would most weaken the conclusion above?

(A) The dental researchers could not discover why toothbrush contamination usually occurred only after toothbrushes had been used for four weeks.
(B) The dental researchers failed to investigate contamination of toothbrushes by viruses, yeasts, and other pathogenic microorganisms.
(C) The dental researchers found that among people who used toothbrushes contaminated with bacterial that cause pneumonia and strep throat, the incidence of these diseases was no higher than among people who used uncontaminated toothbrushes.
(D) The dental researchers found that people who rinsed their toothbrushes thoroughly in hot water after each use were as likely to have contaminated toothbrushes as were people who only rinsed their toothbrushes hurriedly in cold water after each use.
(E) The dental researchers found that, after six weeks of use, greater length of use of a toothbrush did not correlate with a higher number of bacterial being present.
作者: wangyilin888    时间: 2011-4-27 11:10

16. The presence of microorganisms that produce a toxin cause sweater to turn brownish red, phenomenon known as a red ride. Sea otters do not feed in areas where clams, their main source of food, have become contaminated with this toxin. According to a proposed explanation of the otter’s behavior, the otters sample the clams in a potential feeding area and can taste any toxin in them.
  
Which of the following, if true, would most strongly indicate that the hypothesis described in the last sentence of the passage is not correct?

(A) In some of the area where red tides occur, neither clams nor sea otters are indigenous species.
(B) The presence of sea otters in a given area has a significant effect on which other marine organisms are to be found in that areas.
(C) When seawater in an area unaffected by red tide is artificially dyed brownish red, sea otters do not feed on the clams in that area.
(D) If the clams in a given areas are contaminated with toxin, sea otters move to other areas in search of food.
(E) Although very small amounts of the toxin produced during a red tide are not harmful, large doses can be fatal to animals the size of sea otters.
作者: wangyilin888    时间: 2011-4-27 11:10

17. Since 1945 there have been numerous international confrontations as tense as those that precipitated the Second World War, and yet no large-scale conflict has resulted. To explain this, some argue that fear of enormous destruction such as the Second War produced has had a dramatic deterrent effect.
  
Which of the following, if true, most seriously weakens the deterrent theory mentioned above?

(A) After the First World War, the fear of great future destruction was as intense as it was after the Second World War.
(B) Psychologists have determined that the fear of retaliation tends to temper aggressiveness among human beings.
(C) The Second World War was far less distractive than most people generally believe.
(D) Fear of repeating the levels of destruction that the Second World War produced is as pervasive today as it was forty years ago.
(E) Many of the international confrontations that have occurred since 1945 have involved countries that participated in the Second World War.
作者: wangyilin888    时间: 2011-4-27 11:10

18. A conservation group in the United States is trying to change the long-standing image of bats as frightening creatures. The group contends that bats are feared and persecuted solely because they are shy animals that are active only at night.

Which of the following, if true, would cast the most serious doubt on the accuracy of the group’s contention?
A. Bats are steadily losing natural roosting places such as caves and hollow trees and are thus turning to more developed areas for roosting.
B. Bats are the chief consumers of nocturnal insects and thus can help make their hunting territory more pleasant for humans.
C. Bats are regarded as frightening creatures not only in the United States but also in Europe, Africa, and South America.
D. Raccoons and owls are shy and active only at night; yet they are not generally feared and persecuted.
E. People know more about the behavior of other greatly feared animal species, such as lions, alligators, and greatly feared animal species, such as lions, alligators, and snakes, than they do about the behavior of bats.
作者: wangyilin888    时间: 2011-4-27 11:10

19.(Z13)Healthy lungs produce a natural antibiotic that protects them from infection by routinely killing harmful bacteria on airway surfaces. People with cystic fibroses, however, are unable to fight off such bacteria, even though their lungs produce normal amounts of the antibiotic. Since the fluid on airway surfaces in the lungs of people with cystic fibrosis bas an abnormally high salt concentration, scientists hypothesize that in high salt environments the antibiotic becomes ineffective at killing harmful bacteria.
Which of the following, if it were obtained as an experimental result, would most decisively undermine the scientists’ hypothesis?
A.        Healthy lungs in which the salt concentration of the airway-surface fluid has been substantially increased are able to reestablish their normal salt concentration within a relatively short period of time.
B.        The antibiotic produced by the lungs is effective at killing harmful bacteria even when salt concentrations are below levels typical of healthy lungs.
C.        The salt concentration of the airway-surface fluid in the lungs of people who suffer from cystic fibrosis tends to return to its former high levels after having been reduced to levels typical of healthy lungs.
D.        The lungs of people who suffer from cystic fibrosis are unable to fight off harmful bacteria even when the salt concentration is reduced to levels typical of healthy lungs.
E.        The salt concentration in the airway-surface fluid of people whose lungs produce lower-than-average amounts of the antibiotic is generally much lower than that typical of healthy lungs.
作者: wangyilin888    时间: 2011-4-27 11:11

20. The number of people diagnosed as having a certain intestinal disease has dropped significantly in a rural county this year, as compared to last year. Health officials attribute this decrease entirely to improved sanitary conditions at water-treatment plants, which made for cleaner water this year and thus reduced the incidence of the disease.
Which of the following, if true, would most seriously weaken the health officials' explanation for the lower incidence of the disease?
(A)        Many new water-treatment plants have been built in the last five years in the rural county.
(B)        Bottled spring water has not been consumed in significantly different quantities by people diagnosed as having the intestinal disease, as compared to people who did not contract the disease.
(C)        Because of a new diagnostic technique, many people who until this year would have been diagnosed as having the intestinal disease are now correctly diagnosed as suffering from intestinal ulcers.
(D)        Because of medical advances this year, far fewer people who contract the intestinal disease will develop severe cases of the disease.
(E)        The water in the rural county was brought up to the sanitary standards of the water in neighboring counties 10 years ago.
作者: wangyilin888    时间: 2011-4-27 11:11

21. The price of maple syrup has jumped from 22 dollars a gallon three years ago to 40 dollars a gallon today. It can be concluded that maple-syrup harvesters have been artificially inflating prices and that governmental price regulations are necessary to control rising prices.

Which of the following, if true, casts the most doubt on the conclusion drawn above?

(A) The government already requires maple-syrup harvesters to submit their facilities to licensing by
the health department.
(B) Insect infestation and drought have stunted the growth of syrup-producing maple trees and caused less-abundant syrup harvests.
(C) Maple syrup is produced in rural areas that suffer from high unemployment.
(D) Technological improvements in maple-syrup harvesting have reduced production costs.
(E) Maple-syrup prices have risen many times in the past, though never before at the rate recently observed.
作者: wangyilin888    时间: 2011-4-27 11:11

22. (Z3)Guidebook writer:  I have visited hotels throughout the country and have noticed that in those built before 1930 the quality of the original carpentry work is generally superior to that in hotels built afterward.  Clearly carpenters working on hotels before 1930 typically worked with more skill, care, and effort than carpenters who have worked on hotels built subsequently.

Which of the following, if true, most seriously weakens the guidebook writer’s argument?

A.        The quality of original carpentry in hotels is generally far superior to the quality of original carpentry in other structures, such as houses and stores.
B.        Hotels built since 1930 can generally accommodate more guests than those built before 1930.
C.        The materials available to carpenters working before 1930 were not significantly different in quality from the materials available to carpenters working after 1930.
D.        The better the quality of original carpentry in a building, the less likely that building is to fall into disuse and be demolished.
E.        The average length of apprenticeship for carpenters has declined significantly since 1930.
作者: wangyilin888    时间: 2011-4-27 11:12

23.(Z5) In the late 1980’s, the population of sea otters in the North Pacific Ocean began to decline. Of the two plausible explanations for the decline—increased predation by killer whales or disease—disease is the more likely. After all, a concurrent sharp decline in the populations of seals and sea lions was almost certainly caused by a pollution-related disease, which could have spread to sea otters, whereas the population of killer whales did not change noticeably.

Which of the following, if true, most seriously weakens the reasoning?

A.        Killer whales in the North Pacific usually prey on seals and sea lions but will, when this food source is scarce, seek out other prey.
B.        There is no indication that substantial numbers of sea otters migrated to other locations from the North Pacific in the 1980’s.
C.        Along the Pacific coast of North America in the 1980’s, sea otters were absent from many locations where they had been relatively common in former times.
D.        Following the decline in the population of the sea otters, there was an increase in the population of sea urchins, which are sea otters’ main food source.
E.        The North Pacific populations of seals and sea lions cover a wider geographic area than does the population of sea otters.
作者: wangyilin888    时间: 2011-4-27 11:12

练习题:
1.(Z1-13)In the United States, of the people who moved from one state to another when they retired, the percentage who retired to Florida has decreased by three percentage points over the past ten years.  Since many local businesses in Florida cater to retirees, this decline is likely to have a noticeably negative economic effect on these businesses.

Which of the following, if true, most seriously weakens the argument?

A. Florida attracts more people who move from one state to another when they retire than does any other state.
B. The number of people who move out of Florida to accept employment in other states has increased over the past ten years.
C. There are far more local businesses in Florida that cater to tourists than there are local businesses that cater to retirees.
D. The total number of people who retired and moved to another state for their retirement has increased significantly over the past ten years.
E. The number of people who left Florida when they retired to live in another state was greater last year than it was ten years ago.
作者: wangyilin888    时间: 2011-4-27 11:12

2.(Z3)Kate:  The recent decline in numbers of the Tennessee warbler, a North American songbird that migrates each fall to coffee plantations in South America, is due to the elimination of the dense tree cover that formerly was a feature of most South American coffee plantations.

Scott:  The population of the spruce budworm, the warbler’s favorite prey in North America, has been dropping.  This is a more likely explanation of the warbler’s decline.

Which of the following, if true, most seriously calls Scott’s hypothesis into question?

A.        The numbers of the Baltimore oriole, a songbird that dose not eat budworms but is as dependent on South American coffee plantations as is the Tennessee warbler, are declining.
B.        The spruce-budworm population has dropped because of a disease that can infect budworms but not Tennessee warblers.
C.        The drop in the population of the spruce budworm is expected to be only temporary.
D.        Many Tennessee warbler have begun migrating in the fall to places other than traditional coffee plantations.
E.        Although many North American songbirds have declined in numbers, no other species has experienced as great a decline as has the Tennessee warbler.
作者: wangyilin888    时间: 2011-4-27 11:12

3.(Z4)Proposal:  Carbon dioxide and methane in the atmosphere block the escape of heat into space.  So emission of these “greenhouse” gases contributes to global warming.  In order to reduce global warming, emission of greenhouse gases needs to be reduced.  Therefore, the methane now emitted from open landfills should instead be burned to produce electricity.

Objection:  The burning of methane generates carbon dioxide that is released into the atmosphere.

Which of the following, if true, most adequately counters the objection made to the proposal?

A.        Every time a human being or other mammal exhales, there is some carbon dioxide released into the air.
B.        The conversion of methane to electricity would occur at a considerable distance from the landfills.
C.        The methane that is used to generate electricity would generally be used as a substitute for a fuel that does not produce any greenhouse gases when burned.
D.        Methane in the atmosphere is more effective in blocking the escape of heat from the Earth than is carbon dioxide.
E.        The amount of methane emitted from the landfills could be reduced if the materials whose decomposition produces methane were not discarded, but recycled.
作者: wangyilin888    时间: 2011-4-27 11:13

4.(Z4)It is theoretically possible that bacteria developed on Mars early in its history and that some were carried to Earth by a meteorite.  However, strains of bacteria from different planets would probably have substantial differences in protein structure that would persist over time, and no two bacterial strains on Earth are different enough to have arisen on different planets.  So, even if bacteria did arrive on Earth from Mars, they must have died out.

The argument is most vulnerable to which of the following criticisms?

A.        It fails to establish whether bacteria actually developed on Mars.
B.        It fails to establish how likely it is that Martian bacteria were transported to Earth.
C.        It fails to consider whether there were means other than meteorites by which Martian bacteria could have been carried to Earth.
D.        It fails to consider whether all bacteria now on Earth could have arisen from transported Martian bacteria.
E.        It fails to consider whether there could have been strains of bacteria that originated on Earth and later died out.
作者: wangyilin888    时间: 2011-4-27 11:13

5.(Z5)Insect infestations in certain cotton-growing regions of the world have caused dramatic increase in the price of cotton on the world market. By contrast, the price of soybeans has long remained stable. Knowing that cotton plants mature quickly, many soybean growers in Ortovia plan to cease growing soybeans and being raising cotton instead, thereby taking advantage of the high price of cotton to increase their income significantly, at least over the next several years.

Which of the following, if true, most seriously weakens the plan’s chances for success?

A.        The cost of raising soybeans has increased significantly over the past several year and is expected to continue to climb.
B.        Tests of a newly developed, inexpensive pesticide have shown it to be both environmentally safe and effective against the insects that have infested cotton crops.
C.        In the past several years, there has been no sharp increase in the demand for cotton and for goods made out of cotton.
D.        Few consumers would be willing to pay significantly higher prices for cotton goods than they are now paying.
E.        The species of insect that has infested cotton plants has never been known to attack soybean plants.
作者: wangyilin888    时间: 2011-4-27 11:15

6.(Z13)FastMart, a convenience store chain, is planning to add pancake syrup to the items it sells.  FastMart stores do not have shelf space to stock more than one variety of syrup.  Surveys of FastMart customers indicate that one-fourth of them prefer low-calorie syrup, while three-fourths prefer regular syrup.  Since FastMart’s highest priority is to maximize sales, the obvious strategy for it is to stock regular syrup.

Which of the following, if true, most seriously weakens the argument?

A:        People buying pancake syrup at convenience stores, unlike those buying it at supermarkets, generally buy it only a few times.
B.        People who prefer low-calorie syrup generally use about the same amount of syrup on their pancakes as those who prefer regular syrup.
C.        Regular syrup does not sell for a higher price per unit than low-calorie syrup.
D.        In general, customers who prefer regular syrup will buy low-calorie syrup if regular is unavailable, but those who prefer low-calorie will not buy regular syrup.
E.        Sales of syrup are not expected to account for a large proportion of total dollar sales at the average FastMart store.
作者: wangyilin888    时间: 2011-4-27 11:15

7.(Z10)Certain genetically modified strains of maize produce a natural insecticide that protects against maize-eating insects.  The insecticide occurs throughout the plant, including its pollen.  Maize pollen is dispersed by the wind and often blows onto milkweed plants that grow near maize fields.  Caterpillars of monarch butterflies feed exclusively on milkweed leaves.  When, in experiments, these caterpillars were fed milkweed leaves dusted with pollen from modified maize plants, they died.  Therefore, use of the modified maize inadvertently imperils monarch butterflies.

Which of the following, if true, most seriously weakens the argument?

A.        Per unit of volume, the natural insecticide produced by the genetically modified maize plants is less toxic to insects than are many commercial insecticides commonly used on maize plants.
B.        Standard weed-control practices that have been used by farmers for decades have largely eliminated milkweed plants from certain areas where monarch-butterfly caterpillars were once common.
C.        The experiments showed that the caterpillars were not harmed by contact with the pollen from the genetically modified plants unless they ingested it.
D.        The maize-eating insects that the natural insecticide protects against do not feed on the pollen of the maize plant.
E.        Airborne maize pollen tends to collect on the middle leaves of milkweed plants and monarch caterpillars feed only on the plant’s tender upper leaves.
作者: wangyilin888    时间: 2011-4-27 11:15

8.(Z)In 1983 Argonia’s currency, the argon, underwent a reduction in value relative to the world’s strongest currencies. This reduction resulted in a significant increase in Argonia’s exports over 1982 levels. In 1987 a similar reduction in the value of the argon led to another increase in Argonia’s exports. Faced with the need to increase exports yet again, Argonia’s finance minister has proposed another reduction in the value of the argon.

Which of the following, if true, most strongly supports the prediction that the finance minister’s plan will not result in a significant increase in Argonia’s exports next year?
A.        The value of the argon rose sharply last year against the world’s strongest currencies.
B.        In 1988 the argon lost a small amount of its value, and Aronian exports rose slightly in 1989.
C.        The value of Argonia’s exports was lower last year than it was the year before.
D.        All of Argonia’s export products are made by factories that were operating at full capacity last year, and new factories would take years to build.
E.        Reductions in the value of the argon have almost always led to significant reductions in the amount of goods and services that Argonians purchase from abroad.
作者: wangyilin888    时间: 2011-4-27 11:16

9.(Z14)Eurasian watermilfoil, a weed not native to Frida Lake, has reproduced prolifically since being accidentally introduced there.  In order to eliminate the weed, biologists proposed treating infested parts of the lake with a certain herbicide that is nontoxic for humans and aquatic animals.  However, the herbicide might damage populations of certain rare plant species that the lake contains.  For this reason local officials rejected the proposal.

Which of the following, if true, points out the most serious weakness in the officials’ grounds for rejecting the biologists’ proposal?

A.        The continuing spread of Eurasian watermilfoil in Frida Lake threatens to choke out the lake’s rare plant species.
B.        Because of ecological conditions prevailing in its native habitat, Eurasian watermilfoil is not as dominant there as it is in Frida Lake.
C.        The proliferation of Eurasian watermilfoil in Frida Lake has led to reductions in the populations of some species of aquatic animals.
D.        Although Eurasian watermilfoil could be mechanically removed from Frida Lake, eliminating the weed would take far longer this way than it would using herbicides.
E.        Unless Eurasian watermilfoil is completely eliminated from Frida Lake, it will quickly spread again once herbicide treatments or other control measures cease.
作者: wangyilin888    时间: 2011-4-27 11:16

10.(Z14)In each of the past five years, Barraland’s prison population has increased.  Yet, according to official government statistics, for none of those years has there been either an increase in the number of criminal cases brought to trial, or an increase in the rate at which convictions have been obtained.  Clearly, therefore, the percentage of people convicted of crimes who are being given prison sentences is on the increase.

Which of the following, if true, most seriously weakens the argument?

A.        In Barraland the range of punishments that can be imposed instead of a prison sentence is wide.
B.        Over the last ten years, overcrowding in the prisons of Barraland has essentially been eliminated as a result of an ambitious program of prison construction.
C.        Ten years ago, Barraland reformed its criminal justice system, imposing longer minimum sentences for those crimes for which a prison sentence had long been mandatory.
D.        Barraland has been supervising convicts on parole more closely in recent years, with the result that parole violations have become significantly less frequent.
E.        The number of people in Barraland who feel that crime is on the increase is significantly greater now than it was five years ago.
作者: wangyilin888    时间: 2011-4-27 11:17

11(Z5).During the month of May, crabs arrive on Delaware’s beaches to lay eggs.  Certain shorebirds depend on these eggs for food during their annual spring migration to their northern breeding grounds.  Delaware’s crab population has declined recently.  This decline, however, poses no serious threat to the migrating shorebird populations:  by remaining longer in Delaware, the birds will be able to eat enough crab eggs to allow them to complete their migration.

Which of the following, if true, most seriously weakens the argument?

A.        No other food available to the birds on Delaware’s beaches is as nutritious and accessible as are crab eggs.
B.        The decline in the number of crabs on Delaware’s beaches is due to the harvesting of crabs by humans.
C.        There are more crabs on Delaware’s beaches than in any other area that the migrating birds pass through.
D.        The crabs do not conceal their eggs.
E.        The earlier in the season the shorebirds complete their migration, the more likely they are to breed successfully.
作者: wangyilin888    时间: 2011-4-27 11:17

12.(Z15)A survey of entrepreneurs who started companies last year shows that while virtually all did substantial preparatory research and planning, only half used that work to produce a formal business plan.  Since, on average, the entrepreneurs without formal plans secured the capital they needed in half the time of those with plans, these survey results indicate that, in general, formal plans did not help the entrepreneurs who produced them to secure the capital they needed.

Which of the following, if true, most seriously weakens the argument?

A.        Companies started by entrepreneurs who had used formal business plans to attract investment were on the whole as profitable in their first year as were companies started by entrepreneurs who had not produced such plans.
B.        In surveys of entrepreneurs who have attempted without success to raise sufficient capital, more than half of the respondents indicate that they have produced a formal business plan.
C.        Among the entrepreneurs surveyed, those who did not produce formal business plans sought and received a much larger proportion of their capital from investors with whom they had a long-standing business relationship.
D.        The entrepreneurs surveyed who did not produce a formal business plan spent nearly as much time doing preparatory research and planning as the entrepreneurs who produced plans.
E.        The entrepreneurs who produced business plans generally reported later that the process of writing the plan had increased their confidence that their company would succeed.
作者: wangyilin888    时间: 2011-4-27 11:18

13.(Z18)Manatees, aquatic mammals inhabiting Florida’s rivers and coastal waters, swim close to the surface and are frequently killed in collisions with boats.  To address the problem, boat traffic in manatee-populated waters is being required to maintain very low speeds.  Unfortunately, manatees are unable to hear low-pitched sounds and a boat’s sound lowers in pitch as the boat slows.  Therefore, this approach may in fact make things worse rather than better.

Which of the following, if true, casts most doubt on the conclusion?

A.        The areas where boats would have to maintain low speeds were decided partly on the basis of manatee-population estimates and partly from numbers of reported collisions between manatees and boats.
B.        Because the water hyacinth that manatees feed on grows best in water that is nearly still, water hyacinth beds can be disturbed or damaged by fast-moving boat traffic.
C.        Over the last several decades, boat traffic in Florida’s coastal waters has been increasing almost continuously and now represents the greatest threat to the endangered manatee population.
D.        The sound of a boat engine generally travels much further under water than it does through the air.
E.        When experimenters exposed manatees to the recorded sounds of boats moving at various speeds, the creatures were unable to discern the sounds over normal background noise.
作者: wangyilin888    时间: 2011-4-27 11:18

14.(Z18)Political Advertisement:
Mayor Delmont’s critics complain about the jobs that were lost in the city under Delmont’s leadership.  Yet the fact is that not only were more jobs created than were eliminated, but each year since Delmont took office the average pay for the new jobs created has been higher than that year’s average pay for jobs citywide.  So it stands to reason that throughout Delmont’s tenure the average paycheck in this city has been getting steadily bigger.

Which of the following, if true, most seriously weakens the argument in the advertisement?

A.        The unemployment rate in the city is higher today than it was when Mayor Delmont took office.
B.        The average pay for jobs in the city was at a ten-year low when Mayor Delmont took office.
C.        Each year during Mayor Delmont’s tenure, the average pay for jobs that were eliminated has been higher than the average pay for jobs citywide.
D.        Most of the jobs eliminated during Mayor Delmont’s tenure were in declining industries.
E.        The average pay for jobs in the city is currently lower than it is for jobs in the suburbs surrounding the city.
作者: wangyilin888    时间: 2011-4-27 11:19

15.(Z19)In the nation of Partoria, large trucks currently account for 6 percent of miles driven on Partoria’s roads but are involved in 12 percent of all highway fatalities.  The very largest trucks—those with three trailers—had less than a third of the accident rate of single-and double-trailer trucks.  Clearly, therefore, one way for Partoria to reduce highway deaths would be to require shippers to increase their use of triple-trailer trucks.

Which of the following, if true, most seriously weakens the argument?

A.        Partorian trucking companies have so far used triple-trailer trucks on lightly traveled sections of major highways only.
B.        No matter what changes Partoria makes in the regulation of trucking, it will have to keep some smaller roads off-limits to all large trucks.
C.        Very few fatal collisions involving trucks in Partoria are collisions between two trucks.
D.        In Partoria, the safety record of the trucking industry as a whole has improved slightly over the past ten year
E.        In Partoria, the maximum legal payload of a triple-trailer truck is less than three times the maximum legal payload of the largest of the single-trailer trucks.
作者: wangyilin888    时间: 2011-4-27 11:20

16.(Z19)Rabbits were introduced to Numa Island in the nineteenth century.  Overgrazing by the enormous population of rabbits now menaces the island’s agriculture.  The government proposes to reduce the population by using a virus that has caused devastating epidemics in rabbit populations elsewhere.  There is, however, a chance that the virus will infect the bilby, an endangered native marsupial.  The government’s plan, therefore, may serve the interests of agriculture but will clearly increase the threat to native wildlife.

Which of the following, if true, most seriously weakens the argument?

A.        There is less chance that the virus will infect domestic animals on Numa than that it will infect bilbies.
B.        There are no species of animals on the island that prey on the rabbits.
C.        Overgrazing by rabbits endangers many of the plants on which bilbies feed.
D.        The virus that the government proposes to use has been successfully used elsewhere to control populations of rabbits.
E.        There is no alternative means of reducing the rabbit population that would involve no threat to the bilby.
作者: wangyilin888    时间: 2011-4-27 11:21

17.(Z19)Mice that have been given morphine are very likely to develop blood poisoning because bacteria that normally reside in the intestine typically respond to morphine by migrating into the bloodstream.  However, when mice are given both morphine and the new drug naltrexone, blood poisoning is much less frequent, although it does still occur.  These results provide support for researchers’ prediction that naltrexone will turn out to be toxic to certain types of bacteria.

Which of the following, if discovered to be true, would most seriously weaken the support for the researchers’ prediction?

A.        After being administered to mice, naltrexone does not pass from the bloodstream into the intestine.
B.        Naltrexone inhibits morphine from triggering the migration of intestinal bacteria into the bloodstream.
C.        Mice that have been given naltrexone but not morphine have no greater risk of developing blood poisoning than do mice that have not been given either substance.
D.        The increased risk of blood poisoning is not the only harmful effect on mice of being given morphine.
E.        Conditions other than the presence of intestinal bacteria in the bloodstream can cause blood poisoning in mice.
作者: wangyilin888    时间: 2011-4-27 11:23

18.(Z20)Industrial accidents are more common when some of the people in safety-sensitive jobs have drinking problems than when none do.  Since, even after treatment, people who have had drinking problems are somewhat more likely than other people to have drinking problems in the future, any employer trying to reduce the risk of accidents should bar anyone who has ever been treated for a drinking problem from holding a safety-sensitive job.

Which of the following, if true, most seriously undermines the argument above?

A.        Some companies place employees who are being treated for drinking problems in residential programs and allow them several weeks of paid sick leave.
B.        Many accidents in the workplace are the result of errors by employees who do not hold safety-sensitive jobs.
C.        Workers who would permanently lose their jobs if they sought treatment for a drinking problem try instead to conceal their problem and continue working for as long as possible.
D.        People who hold safety-sensitive jobs are subject to stresses that can exacerbate any personal problems they may have, including drinking problems.
E.        Some industrial accidents are caused by equipment failure rather than by employee error.
作者: wangyilin888    时间: 2011-4-27 11:24

19.(Z20)A major chemical spill occurred five years ago at Baker’s Beach, the world’s sole nesting ground for Merrick sea turtles, and prevented nearly all the eggs laid that year from hatching.  Yet the number of adult female Merricks returning to lay their eggs at Baker’s Beach has actually increased somewhat since five years ago.  Clearly, environmentalists’ prediction that the world’s Merrick population would decline as a result of the spill has proven unfounded.

Which of the following, if true, most seriously undermines the argument offered in refutation of the environmentalists’ prediction?

A.        The chemical spill five years ago occurred at a time when there were neither Merrick sea turtles nor Merrick sea turtle eggs on Baker’s Beach.
B.        Female Merrick sea turtles begin returning to Baker’s Beach to lay their eggs when they are ten years old.
C.        Under normal conditions, only a small proportion of hatchling female Merrick sea turtles survive in the ocean until adulthood and return to lay their eggs at Baker’s Beach.
D.        Environmental pressures unrelated to the chemical spill have caused a significant decline in the population of one of the several species of sea birds that prey on Merrick sea turtle eggs.
E.        After the chemical spill, an environmental group rejected a proposal to increase the Merrick sea turtle population by transferring eggs from Baker’s Beach to nearby beaches that had not been affected by the spill.

Secret passwords are often used to control access to computers. When emoployees
As their initials or birth dates. To improve security, employers should assign ran-
domly generaied passwords to employees rather than allowing employees to make up their own
作者: wangyilin888    时间: 2011-4-27 11:26

20.(Z21)Which of the following, if true, most seriously undermines the conclusiun drawm above?

A.        if passwords are generated randomly, it is theoretically possible that employees will be assigned passwords that they might have selected on their own.
B.        Randomly generated passwords are so difficult for employees to recall that they often record the passwords in places where the passwords could be easily seen by others.
C.        Computer systems protected by passwords are designed to ignore commands that are entered by employees or others who use invalid passwords.
D.        In general, the higher the level of security maintained at the computer system, the more difficult it is for unauthorized users to obtain access to the system.
E.        Control of physical access to computers by the use of locked doors and guards should be used un addition to passwords in order to maintain security.
作者: wangyilin888    时间: 2011-4-27 11:26

21.(Z21)In recent years, networks of fiber-optic cable have been replacing electrical wire for transmitting telecommunications signals, Signals running through fiber-optic cables deteriorate, and wo the signals must be run through a piece of e3quiplent called a regenerator before being transmitted father. Light-Line is the leading manufacturer of signal regenerators. Therefore, Light-line is one of the companies that will most benefit if new long-distance fiber-optic telecommunications networks are constructed.

Which of the following, if true, casts the most doubt on the prediction about light-ling’sprospects?

A. Telecommunications signals sent along electrical wires can travel much Farther than signals transmitted through fiber-optic before needing to be
Regenerated.
B. Expanding its production of regenerators will require Light-line to spend proportionately more on labor and materials.
C. The volume of signals that a fiber-optic cable can carry is several Thousand times greater than the volume that can be carried by an electrical Wire of similar size.
D. There are technologies that enable telecommunications signals to be transmil
Without either wire or fiber-optic cable

F.        Several manufacturers are developing regenerator technologies that will allow to be transmitted many times farther than at present before requiring regeneration.
作者: wangyilin888    时间: 2011-4-27 11:26

22.(Z21)Rail Executive: Five years age we discontinued train service between Lamberton and its suburbs because low ridership caused total fares collected to be substantially lower than the cost of operating the service it is true that recent population growth in the suburban communities suggests increased potential ridership, neverthelesssince most of the newer residents own automobiles, restoring the train service this year would still result in serious deficits

Which of the following ,if true casts the most serious doubt on the rail executive’s argument?

A. Once train service along a given route has been discontinued ,there are substantial start-up costs associated with restoring service along that route
B. Most of the new residents of Lamberton’s suburbs do not need to
travel into Lamberton regularly for shopping or recreation
C. If train service were restored this with increased hours of service there would not be an increase mcrease in total ridership
D. A survey conducted last year showed that few people living in Lamberton or its suburbs support increased public funding for mass transportation.
E. Recently there have been sharp increases in parking fees in Lamberton and in highway tolls on major commuter routes.
作者: wangyilin888    时间: 2011-4-27 11:27

23.(Z21)A shortage of orders for Manto Aircraft’s airliners has led analysts to predict that the manufacturer will have to lay workers ,the new order that consolidated airlines has just made for 20 of Manto’s model TX jets does not provide a reason for the analysts to revise their predictions ,because simultaneously with its new order , consolidated canceled its existing order for an equal number of manto’s larger, more expensive model Z uets

which of the following ,if true, most seriously weakens the argument?

A. Manto relies more heavily on outside subcontractors to provide the labor to
manufacture the Model Z than it does the Model TX

B. The Manto employees who currently work to manufacture the model Z are
Not proficient at manufacturing the model TX
C. Manto includes all required maintenance work for the first five years in the
price of each jet it sells .
D. Manto has had to lay off workers several times within the past ten years but
has typically rehired many of the workers when it subsequently received new orders
E. A large number of the airliners in consolidated is fleet are at the beginning of
their expected service life
作者: wangyilin888    时间: 2011-4-27 11:27

24.(Z22)Travel agents are market intermediaries who make their living by gathering, organizing,and dispensing information about travel-related services
that is not readily available to most consumers through new information
technologies,such as the internet much of this information can now be made directly available to consumers demand for the services of travel agents will be drastically reduced

Which of the following, if true, most seriously weakens the argument?

A. Travel agents routinely use the internet and other new information tech-
nologies as sources for the information they obtain for their customers .
B. The amount of information available through the internet and other new information technologies is increasing faster than the capabilities of most consumers to process it
C. Many people use travel-related services, such as airlines and hotels without consulting a travel agent .
D. The people who currently use the services of travel agents are also those most likely to gain access to new information technologies
E. The internet and other new information technologies are currently used by a relatively small proportion of the population
作者: wangyilin888    时间: 2011-4-27 11:27

25.(Z22)in recent years, networks of fiber-optic cable have been replacing electrical wire for transmitting telecommunications signals, Signals running through fiber-optic cables deteriorate, and wo the signals must be run through a piece of e3quiplent called a regenerator before being transmitted father. Light-Line is the leading manufacturer of signal regenerators. Therefore, Light-line is one of the companies that will most benefit if new long-distance fiber-optic telecommunications networks are constructed.

Which of the following, if true, casts the most doubt on the prediction about light-ling’s prospects?

A. Telecommunication’s signals sent along electrical wires can travel much Farther than signals transmitted through fiber-optic before needing to be regenerated.
B. Expanding its production of regenerators will require Light-line to spend proportionately more on labor and materials.
C. The volume of signals that a fiber-optic cable can carry is several Thousand times greater than the volume that can be carried by an electrical Wire of similar size.
D. There are technologies that enable telecommunications signals to be transmitted without either wire or fiber-optic cable
E. Several manufacturers are developing regenerator technologies that willow signals to be transmitted many times farther than at present before requiring regeneration
作者: wangyilin888    时间: 2011-4-27 11:28

3. 加强题
加强题标志词:


加强题类型:





加强题做题方法:





例题:
1. Because incumbent members of Congress are given a great deal of attention by the news media and because they enjoy such perquisites as free mail privileges and generous travel allowances, incumbents enjoy an overwhelming advantage over their challengers in elections for the United States Congress.
  
Which of the following, if true, best supports the claim above?

(A) In the last congressional elections, incumbents met with a larger number of lobbyists than did challengers.
(B) In the last congressional elections, 98 percent of the incumbents in the House of Representatives who were seeking reelection won.
(C) Incumbent members of Congress are frequently critical of the amount of attention given to them by the news media.
(D) The support that political station action committees provide to challengers for congressional seats often compensates for the perquisites enjoyed by incumbent members of Congress.
(E) Of all incumbent senators surveyed before the last congressional elections, 78 percent said that their challengers did not pose a serious threat to their chances for reelection.
作者: wangyilin888    时间: 2011-4-27 11:28

2. A cost-effective solution to the problem of airport congestion is to provide high-speed ground transportation between major cities lying 200 to 500 miles apart. The successful implementation of this plan would cost far less than expanding existing airports and would also reduce the number of airplanes clogging both airports and airways.

Which of the following, if true, could be proponents of the plan above most appropriately cite as a piece of evidence for the soundness of their plan?
A. An effective high-speed ground-transportation system would require major repairs to many highways and mass-transit improvements.
B. One-half of all departing flights in the nation’s busiest airport head for a destination in a major city 225 miles away.
C. The majority of travelers departing from rural airports are flying to destinations in cities over 600 miles away.
D. Many new airports are being built in areas that are presently served by high-speed ground-transportation systems.
E. A large proportion of air travelers are vacationers who are taking long-distance flights.
作者: wangyilin888    时间: 2011-4-27 11:29

3. Recent surveys show that many people who seek media help are under a great deal of stress. Medical research also shows that stress can adversely affect an individual’s immune system, which is responsible for combating many infections. Thus when a person is under stress, he or she is more likely to become ill.
  
Which of the following, if true, would most strength the conclusion above?

(A) Many businesses that provide health insurance for their employees also provide seminars on stress management.
(B) Many businesses report a significant decrease in absenteeism during periods when employees feel pressured by management.
(C) There is a marked decrease in the number of complaints presented at college infirmaries during vacation time.
(D) There is a marked increase in the number of illness treated at college infirmaries around the time of examinations.
(E) Most people report that being in a hospital or an infirmary is a stressful situation.
作者: wangyilin888    时间: 2011-4-27 11:29

4. If the airspace around centrally located airports were restricted to commercial airliners and only those private planes equipped with radar, most of the private-plane traffic would be forced to sue outlying airfields. Such a reduction in the amount of private-plane traffic would reduce the risk of midair collision around the centrally located airports.

Which of the following, if true, would most strengthen the conclusion drawn in the second sentence?

A. Commercial airliners are already required by law to be equipped with extremely sophisticated radar systems.
B. Centrally located airports are experiencing overcrowded airspace primarily because f sharp increases in commercial-airline traffic.
C. Many pilots of private planes would rather buy radar equipment than be excluded from centrally located airports.
D. The number of midair collisions that occur near centrally located airports has decreased in recent years.
E. Private planes not equipped with radar systems cause a disproportionately large number of midair collisions around centrally located airports.
作者: wangyilin888    时间: 2011-4-27 11:30

5.(Z)In parts of South America, vitamin-A deficiency is a serious health problem, especially among children.  In one region, agriculturists are attempting to improve nutrition by encouraging farmers to plant a new variety of sweet potato called SPK004 that is rich in beta-carotene, which the body converts into vitamin A.  The plan has good chances of success, since sweet potato is a staple of the region’s diet and agriculture, and the varieties currently grown contain little beta-carotene.

Which of the following, if true, most strongly supports the prediction that the plan will succeed?

A.        The growing conditions required by the varieties of sweet potato currently cultivated in the region are conditions in which SPK004 can flourish.
B.        The flesh of SPK004 differs from that of the currently cultivated sweet potatoes in color and texture, so traditional foods would look somewhat different when prepared from SPK004.
C.        There are no other varieties of sweet potato that are significantly richer in beta-carotene than SPK004 is.
D.        The varieties of sweet potato currently cultivated in the region contain some important nutrients that are lacking in SPK004.
E.        There are other vegetables currently grown in the region that contain more beta-carotene than the currently cultivated varieties of sweet potato do.
作者: wangyilin888    时间: 2011-4-27 11:30

6.(Z)The spacing of the four holes on a fragment of a bone flute excavated at a Neanderthal campsite is just what is required to play the third through sixth notes of the diatonic scale—the seven-note musical scale used in much of Western music since the Renaissance.  Musicologists therefore hypothesize that the diatonic musical scale was developed and used thousands of years before it was adopted by Western musicians.

Which of the following, if true, most strongly supports the hypothesis?

A.        Bone flutes were probably the only musical instrument made by Neanderthals.
B.        No musical instrument that is known to have used a diatomic scale is of an earlier date than the flute found at the Neanderthal campsite.
C.        The flute was made from a cave-bear bone and the campsite at which the flute fragment was excavated was in a cave that also contained skeletal remains of cave bears.
D.        Flutes are the simplest wind instrument that can be constructed to allow playing a diatonic scale.
E.        The cave-bear leg bone used to make the Neanderthal flute would have been long enough to make a flute capable of playing a complete diatonic scale.
作者: wangyilin888    时间: 2011-4-27 11:30

7.(Z)Studies in restaurants show that the tips left by customers who pay their bill in cash tend to be larger when the bill is presented on a tray that bears a credit-card logo.  Consumer psychologists hypothesize that simply seeing a credit-card logo makes many credit-card holders willing to spend more because it reminds them that their spending power exceeds the cash they have immediately available.

Which of the following, if true, most strongly supports the psychologists’ interpretation of the studies?

A.        The effect noted in the studies is not limited to patrons who have credit cards.
B.        Patrons who are under financial pressure from their credit-card obligations tend to tip less when presented with a restaurant bill on a tray with credit-card logo than when the tray has no logo.
C.        In virtually all of the cases in the studies, the patrons who paid bills in cash did not possess credit cards.
D.        In general, restaurant patrons who pay their bills in cash leave larger tips than do those who pay by credit card.
E.        The percentage of restaurant bills paid with given brand of credit card increases when that credit card’s logo is displayed on the tray with which the bill is prepared.
作者: wangyilin888    时间: 2011-4-27 11:30

8.(Z)Studies in restaurants show that the tips left by customers who pay their bill in cash tend to be larger when the bill is presented on a tray that bears a credit-card logo.  Consumer psychologists hypothesize that simply seeing a credit-card logo makes many credit-card holders willing to spend more because it reminds them that their spending power exceeds the cash they have immediately available.

Which of the following, if true, most strongly supports the psychologists’ interpretation of the studies?

A.        The effect noted in the studies is not limited to patrons who have credit cards.
B.        Patrons who are under financial pressure from their credit-card obligations tend to tip less when presented with a restaurant bill on a tray with credit-card logo than when the tray has no logo.
C.        In virtually all of the cases in the studies, the patrons who paid bills in cash did not possess credit cards.
D.        In general, restaurant patrons who pay their bills in cash leave larger tips than do those who pay by credit card.
E.        The percentage of restaurant bills paid with given brand of credit card increases when that credit card’s logo is displayed on the tray with which the bill is prepared.
作者: wangyilin888    时间: 2011-4-27 11:31

9.(Z)It is true of both men and women that those who marry as young adults live longer than those who never marry.  This does not show that marriage causes people to live longer, since, as compared with other people of the same age, young adults who are about to get married have fewer of the unhealthy habits that can cause a person to have a shorter life, most notably smoking and immoderate drinking of alcohol.

Which of the following, if true, most strengthens the argument above?

A. Marriage tends to cause people to engage less regularly in sports that involve risk of bodily harm.
B. A married person who has an unhealthy habit is more likely to give up that habit than a person with the same habit who is unmarried.
C. A person who smokes is much more likely than a nonsmoker to marry a person who smokes at the time of marriage, and the same is true for people who drink alcohol immoderately.
D. Among people who marry as young adults, most of those who give up an unhealthy habit after marriage do not resume the habit later in life.
E.Among people who as young adults neither drink alcohol immoderately nor smoke, those who never marry live as long as those who marry.
作者: wangyilin888    时间: 2011-4-27 11:31

10. The population of peregrine falcons declined rapidly during the 1950’s and 1960’s and reached an all-time low in the early 1970’s. The decline was attributed by scientists to the widespread use of the pesticide DDT in rural areas.
  
Which of the following, if true, gives the strongest support to the scientists claim?

(A) DDT was not generally in use in areas devoted to heavy industry.
(B) In the time since the use of DDT was banned in 1972, the population of peregrine falcons has been steadily increasing.
(C) Peregrine falcons, like other birds of prey, abandon eggs that fallen out of the nest, even if the eggs remain intact.
(D) Starling, house sparrows, and blue jays-birds the peregrine falcon prey on—were not adversely affected by DDT in their habitats.
(E) Other birds of prey, such as the osprey, the bald cage, and the brown pclican, are found in the same area as is the peregrine falcon.
作者: wangyilin888    时间: 2011-4-27 11:31

11.(Z)Healthy lungs produce a natural antibiotic that protects them from infection by routinely killing harmful bacteria on airway surfaces.  People with cystic fibrosis, however, are unable to fight off such bacteria, even though their lungs produce normal amounts of the antibiotic.  The fluid on airway surfaces in the lungs of people with cystic fibrosis has an abnormally high salt concentration; accordingly, scientists hypothesize that the high salt concentration is what makes the antibiotic ineffective.

Which of the following, if true, most strongly supports the scientists’ hypothesis?

A.        When the salt concentration of the fluid on the airway surfaces of healthy people is raised artificially, the salt concentration soon returns to normal.
B.        A sample of the antibiotic was capable of killing bacteria in an environment with an unusually low concentration of salt.
C.        When lung tissue from people with cystic fibrosis is maintained in a solution with a normal salt concentration, the tissue can resist bacteria.
D.        Many lung infections can be treated by applying synthetic antibiotics to the airway surfaces.
E.        High salt concentrations have an antibiotic effect in many circumstances.
作者: wangyilin888    时间: 2011-4-27 11:31

12.(Z)Political Advertisement:
Mayor Delmont’s critics complain about the jobs that were lost in the city under Delmont’s leadership.  Yet the fact is that not only were more jobs created than were eliminated, but the average pay for these new jobs has been higher than the average pay for jobs citywide every year since Delmont took office.  So there can be no question that throughout Delmont’s tenure the average paycheck in this city has been getting steadily bigger.

Which of the following, if true, most strengthens the argument in the advertisement?

A.        The average pay for jobs created in the city during the past three years was higher than the average pay for jobs created in the city earlier in Mayor Delmont’s tenure.
B.        Average pay in the city was at a ten-year low when Mayor Delmont took office.
C.        Some of the jobs created in the city during Mayor Delmont’s tenure have in the meantime been eliminated again.
D.        The average pay for jobs eliminated in the city during Mayor Delmont’s tenure has been roughly equal every year to the average pay for jobs citywide.
E.        The average pay for jobs in the city is currently higher than it is for jobs in the suburbs surrounding the city.
作者: wangyilin888    时间: 2011-4-27 11:32

13.(Z)Capuchin monkeys often rub their bodies with a certain type of millipede.  Laboratory tests show that secretions from the bodies of these millipedes are rich in two chemicals that are potent mosquito repellents, and mosquitoes carry parasites that debilitate capuchins.  Some scientists hypothesize that the monkeys rub their bodies with the millipedes because doing so helps protect them from mosquitoes.

Which of the following, if true, provides the most support for the scientists’ hypothesis?

A.        A single millipede often gets passed around among several capuchins, all of whom rub their bodies with it.
B.        The two chemicals that repel mosquitoes also repel several other varieties of insects.
C.        The capuchins rarely rub their bodies with the millipedes except during the rainy season, when mosquito populations are at their peak.
D.        Although the capuchins eat several species of insects, they do not eat the type of millipede they use to rub their bodies.
E.        The two insect-repelling chemicals in the secretions of the millipedes are carcinogenic for humans but do not appear to be carcinogenic for capuchins.
作者: wangyilin888    时间: 2011-4-27 11:32

14.(Z)Crowding on Mooreville’s subway frequently leads to delays, because it is difficult for passengers to exit from the trains.  Subway ridership is projected to increase by 20 percent over the next 10 years.  The Metroville Transit Authority plans to increase the number of daily train trips by only 5 percent over the same period.  Officials predict that this increase is sufficient to ensure that the incidence of delays due to crowding does not increase.

Which of the following, if true, provides the strongest grounds for the officials’ prediction?

A.        By changing maintenance schedules, the Transit Authority can achieve the 5 percent increase in train trips without purchasing any new subway cars.
B.        The Transit Authority also plans a 5 percent increase in the number of bus trips on routes that connect to subways.
C.        For most commuters who use the subway system, there is no practical alternative public transportation available.
D.        Most of the projected increase in ridership is expected to occur in off-peak hours when trains are now sparsely used.
E.        The 5 percent increase in the number of train trips can be achieved without an equal increase in Transit Authority operational costs.
作者: wangyilin888    时间: 2011-4-27 11:32

15.(Z)Certain politicians in the country of Birangi argue that a 50 percent tax on new automobiles would halt the rapid increase of automobiles on Birangi’s roads and thereby slow the deterioration of Birangi’s air quality.  Although most experts agree that such a tax would result in fewer Birangians buying new vehicles and gradually reduce the number of automobiles on Birangi’s roads, they contend that it would have little impact on Birangi’s air-quality problem.

Which of the following, if true in Birangi, would most strongly support the experts’ contention about the effect of the proposed automobile tax on Birangi’s air-quality problem?

A.        Automobile emissions are the largest single source of air pollution.
B.        Some of the proceeds from the new tax would go toward expanding the nonpolluting commuter rail system.
C.        Currently, the sales tax on new automobiles is considerably lower than 50 percent.
D.        Automobiles become less fuel efficient and therefore contribute more to air pollution as they age.
E.        The scrapping of automobiles causes insignificant amounts of air pollution.
作者: wangyilin888    时间: 2011-4-27 11:33

16.(Z)A mosquito bite can transmit to a person the parasite that causes malaria, and the use of mosquito nets over children’s beds can significantly reduce the incidence of malarial infection for children in areas where malaria is common. Yet public health officials are reluctant to recommend the use of mosquito nets over children’s beds in such areas.

Which of the following, if true, would provide the strongest grounds for the public health officials’ reluctance?

A.        Early exposure to malaria increases the body’s resistance to it and results in a lesser likelihood of severe life-threatening episodes of malaria.
B.        Mosquito bites can transmit to people diseases other than malaria.
C.        Mosquito nets provide protection from some insect pests other than mosquitoes.
D.        Although there are vaccines available for many childhood diseases, no vaccine has been developed that is effective against malaria.
E.        The pesticides that are most effective against mosquitoes in regions where malaria is common have significant detrimental effects on human health.
作者: wangyilin888    时间: 2011-4-27 11:35

17.(Z)The Eurasian ruffe, a fish species inadvertently introduced into North America’s Great Lakes in recent years, feeds on the eggs of lake whitefish, a native species, thus threatening the lakes’ natural ecosystem. To help track the ruffe’s spread, government agencies have produced wallet-sized cards about the ruffe. The cards contain pictures of the ruffe and explain the danger they pose; the cards also request anglers to report any ruffe they catch.

Which of the following, if true, would provide most support for the prediction that the agencies’ action will have its intended effect?

A.        The ruffe has spiny fins that make it unattractive as prey.
B.        Ruffe generally feed at night, but most recreational fishing on the Great Lakes is done during daytime hours.
C.        Most people who fish recreationally on the Great Lakes are interested in the preservation of the lake whitefish because it is a highly prized game fish.
D.        The ruffe is one of several nonnative species in the Great Lakes whose existence threatens the survival of lake whitefish populations there.
E.        The bait that most people use when fishing for whitefish on the Great Lakes is not attractive to ruffe.
作者: wangyilin888    时间: 2011-4-27 11:35

18.(Z)The traditional treatment of strep infections has been a seven-day course of antibiotics, either penicillin or erythromycin. However, since many patients stop taking those drugs within three days, reinfection is common in cases where these drugs are prescribed. A new antibiotic requires only a three-day course of treatment. Therefore, reinfection will probably be less common in cases where the new antibiotic is prescribed than in cases where either penicillin or erythromycin is prescribed.

Which of the following, if true, most strengthens the argument?
Tricky!!
A.        Some of people who are allergic to penicillin are likely to be allergic to the new antibiotic.
B.        A course of treatment with the new antibiotic costs about the same as a course of treatment with either penicillin or erythromycin.
C.        The new antibiotic has been shown to be effective in eradicating bacterial infections other than strep.
D.        Some physicians have already begun to prescribe the new antibiotic instead of penicillin or erythromycin for the treatment of some strep infections
E.        Regardless of whether they take a traditional antibiotic or the new one, most patients feel fully recovered after taking the drug for three days.
作者: wangyilin888    时间: 2011-4-27 11:36

19.(Z)In response to viral infection, the immune systems of mice typically produce antibodies that destroy the virus by binding to proteins on its surface.  Mice infected with a herpesvirus generally develop keratitis, a degenerative disease affecting part of the eye.  Since proteins on the surface of cells in this part of the eye closely resemble those on the herpesvirus surface, scientists hypothesize that these cases of keratitis are caused by antibodies to herpesvirus.

Which of the following, if true, gives the greatest additional support to the scientists’ hypothesis?

A.        Other types of virus have surface proteins that closely resemble proteins found in various organs of mice.
B.        There are mice that are unable to form antibodies in response to herpes infections, and these mice contract herpes at roughly the same rate as other mice.
C.        Mice that are infected with a herpesvirus but do not develop keratitis produce as many antibodies as infected mice that do develop keratitis.
D.        There are mice that are unable to form antibodies in response to herpes infections, and these mice survive these infections without ever developing keratitis.
E.        Mice that have never been infected with a herpesvirus can sometimes develop keratitis.
作者: wangyilin888    时间: 2011-4-27 11:36

20.(Z) Lyme disease is caused by a bacterium transmitted to humans by deer ticks. Generally deer ticks pick up the bacterium while in the larval stage from feeding on infected white-footed mice. However, certain other species on which the larvae feed do not harbor the bacterium. Therefore, if the population of these other species were increased, the number of ticks acquiring the bacterium and hence the number of people contracting Lyme disease would likely decline.

Which of the following, if true, most strengthens the argument?

A.        Ticks do not suffer any adverse consequences from carrying the bacterium that causes Lyme disease in humans.
B.        There are no known cases of a human’s contracting Lyme disease through contact with white-footed mice.
C.        A deer tick feeds only once while in the larval stage.
D.        A single host animal can be the source of bacterium for many tick larvae.
E.        None of the other species on which deer tick larvae feed harbor other bacteria that ticks transmit to humans
作者: wangyilin888    时间: 2011-4-27 11:36

练习题:
1. The town of Stavanger, Norway, was quiet and peaceful until early 1960’s, when Stavanger became Norway’s center for offshore oil exploration. Between then and now, violent crime and vandalism in Stavanger have greatly increased. Clearly, these social problems are among the results of Stavanger’s oil boom.
  
Which of the following, if it occurred between the early 1960’s and now, gives the strongest support to the argument above?

(A) The people of Stavanger rarely regret that their town was chosen to be Norway’s center for offshore oil exploration.
(B) Norwegian sociologists expressed grave concern about the increase in violent crime and vandalism in Stavanger.
(C) Violent crime and vandalism have remained low in Norwegian that had no oil boom.
(D) Nonviolent crime, drug addiction, and divorce in Stavanger increased approximately as much as violent crime and vandalism did.
(E) The oil boom necessitated the building of wider roads for the increased traffic in Stavanger.
作者: wangyilin888    时间: 2011-4-27 11:37

2. Airline: Newly developed collision-avoidance systems, although not fully tested to discover potential malfunctions, must be installed immediately in passenger planes. Their mechanical warnings enable pilots to avoid crashes.

Pilots: Pilots will not fly in planes with collision-avoidance systems that are not fully tested. Malfunctioning systems could mislead pilots, causing crashes.

The pilots’ objection is most strengthened if which of the following is true?
(A) It is always possible for mechanical devices to malfunction.
(B) Jet engines, although not fully tested when first put into use, have achieved exemplary performance and safety records.
(C) Although collision-avoidance systems will enable pilots to avoid some crashes, the likely malfunctions of the not-fully-tested systems will cause even more crashes.
(D) Many airline collisions are caused in part by the exhaustion of overworked pilots.
(E) Collision-avoidance systems, at this stage of development, appear to have worked better in passenger planes than in cargo planes during experimental flights made over a six-month period.
作者: wangyilin888    时间: 2011-4-27 11:37

3. Two decades after the Emerald River Dam was built, none of the eight fish species native to the Emerald River was still reproducing adequately in the river below the dam. Since the dam reduced the annual range of water temperature in the river below the dam from 50 degrees to 6 degrees, scientists have hypothesized that sharply rising water temperatures must be involved in signaling the native species to begin the reproductive cycle.

Which of the following statements, if true, would most strengthen the scientists’ hypothesis?

(A) The native fish species were still able to reproduce only in side streams of the river below the dam where the annual temperature range remains approximately 50 degrees.
(B) Before the dam was built, the Emerald River annually overflowed its banks, creating backwaters that were critical breeding areas for the native species of fish.
(C) The lowest recorded temperature of the Emerald River before the dam was built was 34 degrees, whereas the lowest recorded temperature of the river after the dam was built has been 43 degrees.
(D)Nonnative species of fish, introduced into the Emerald River after the dam was built, have begun competing with the declining native fish species for food and space.
(E) Five of the fish species native to the Emerald River are not native to any other river in North America.
作者: wangyilin888    时间: 2011-4-27 11:37

4. In recent years many cabinetmakers have been winning acclaim as artists. But since furniture must be useful, cabinetmakers must exercise their craft with an eye to the practical utility of their product. For this reason, cabinetmaking is not art.

Which of the following is an assumption that supports drawing the conclusion above from the reason given for that conclusion?

(A) Some furniture is made to be placed in museums, where it will not be used by anyone.
(B) Some cabinetmakers are more concerned than others with the practical utility of the products they produce.
(C) Cabinetmakers should be more concerned with the practical utility of their products than they currently are.
(D) An object is not an art object if its maker pays attention to the object’s practical utility.
(E) Artists are not concerned with the monetary value of their products.
作者: wangyilin888    时间: 2011-4-27 11:38

5. Since the routine use of antibiotics can give rise to resistant bacteria capable of surviving antibiotic environments, the presence of resistant bacteria in people could be due to the human use of prescription antibiotics. Some scientists, however, believe that most resistant bacteria in people derive from human consumption of bacterially infected meat.

Which of the following statements, if true, would most significantly strengthen the hypothesis of the scientists?

(A) Antibiotics are routinely included in livestock feed so that livestock producers can increase the rate of growth of their animals.
(B) Most people who develop food poisoning from bacterially infected meat are treated with prescription antibiotics.
(C) The incidence of resistant bacteria in people has tended to be much higher in urban areas than in rural areas where meat is of comparable quality.
(D) People who have never taken prescription antibiotics are those least likely to develop resistant bacteria.
(E) Livestock producers claim that resistant bacteria in animals cannot be transmitted to people through infected meat.
作者: wangyilin888    时间: 2011-4-27 11:38

6. Male bowerbirds construct elaborately decorated nests, or bowers. Basing their judgment on the fact that different local populations of bowerbirds of the same species build bowers that exhibit different building and decorative styles, researchers have concluded that the bowerbirds’ building styles are a culturally acquired, rather than a genetically transmitted, trait.

Which of the following, if true, would most strengthen the conclusion drawn by the researchers?

(A) There are more common characteristics than there are differences among the bower-building styles of the local bowerbird population that has been studied most extensively
(B) Young male bowerbirds are inept at bower-building and apparently spend years watching their elders before becoming accomplished in the local bower style.
(C) The bowers of one species of bowerbird lack the towers and ornamentation characteristic of the bowers of most other species of bowerbird.
(D) Bowerbirds are found only in New Guinea and Australia, where local populations of the birds apparently seldom have contact with one another.
(E) It is well known that the song dialects of some songbirds are learned rather than transmitted genetically.
作者: wangyilin888    时间: 2011-4-27 11:38

7. Many breakfast cereals are fortified with vitamin supplements. Some of these cereals provide 100 percent of the recommended daily requirement of vitamins. Nevertheless, a well-balanced breakfast, including a variety of foods, is a better source of those vitamins than are such fortified breakfast cereals alone.

Which of the following, if true, would most strongly support the position above?

(A) In many foods, the natural combination of vitamins with other nutrients makes those vitamins more usable by the body than are vitamins added in vitamin supplements.
(B) People who regularly eat cereals fortified with vitamin supplements sometimes neglect to eat the foods in which the vitamins occur naturally.
(C)Foods often must be fortified with vitamin supplements because naturally occurring vitamins are removed during processing.
(D) Unprocessed cereals are naturally high in several of the vitamins that are usually added to fortified breakfast cereals.
(E) Cereals containing vitamin supplements are no harder to digest than similar cereals without added vitamins.




欢迎光临 国际顶尖MBA申请交流平台--TOPWAY MBA (http://forum.topway.org/) Powered by Discuz! 7.2